PT4.S1.Q8 - Political theorist: The chief foundations of all

MambaMentality93MambaMentality93 Live Member
edited March 2019 in Logical Reasoning 503 karma

Hey guys,

I'm having trouble understanding question 8 from PT4 section 1. I diagrammed it:
-PWP ---> -GLS and realized that the passage makes an illegal reverse. It should have been GLS ---> PWP (the contrapositive) instead of PWP---> GLS. However, even with that in mind, I wasn't able to find the right answer. Now, I don't understand why D is the right answer. Can someone please help me?

Cheers!

Comments

  • FixedDiceFixedDice Member
    1804 karma

    The problem with the stimulus is not that it makes a wrongful reversal, but that it makes a wrongful negation. You are kind of right, however, as those two errors tend to go hand-in-hand: both wrongful reversal and wrongful negation assume that A ---> B and B ---> A.

    The stimulus says that, since ~PWP ---> ~GLS, PWP ---> GLS. The contrapositive of the stimulus is GLS ---> PWP, however. So the argument assumes its own conclusion without justification. This is what (D) points out: the argument assumes PWP is sufficient for GLS.

  • crazycalmcrazycalm Free Trial Member
    157 karma

    Hi!

    You're right that the conclusion is an illegal reversal. By saying PWP --> GLS, the conclusion mistakenly states that PWP is the sufficient condition and that GLS is the necessary condition. (D) reflects this.

  • MambaMentality93MambaMentality93 Live Member
    503 karma

    Thank you so much mates!!!!!

Sign In or Register to comment.